-
Notifications
You must be signed in to change notification settings - Fork 1
/
Jordan.tex
1231 lines (1037 loc) · 41.9 KB
/
Jordan.tex
1
2
3
4
5
6
7
8
9
10
11
12
13
14
15
16
17
18
19
20
21
22
23
24
25
26
27
28
29
30
31
32
33
34
35
36
37
38
39
40
41
42
43
44
45
46
47
48
49
50
51
52
53
54
55
56
57
58
59
60
61
62
63
64
65
66
67
68
69
70
71
72
73
74
75
76
77
78
79
80
81
82
83
84
85
86
87
88
89
90
91
92
93
94
95
96
97
98
99
100
101
102
103
104
105
106
107
108
109
110
111
112
113
114
115
116
117
118
119
120
121
122
123
124
125
126
127
128
129
130
131
132
133
134
135
136
137
138
139
140
141
142
143
144
145
146
147
148
149
150
151
152
153
154
155
156
157
158
159
160
161
162
163
164
165
166
167
168
169
170
171
172
173
174
175
176
177
178
179
180
181
182
183
184
185
186
187
188
189
190
191
192
193
194
195
196
197
198
199
200
201
202
203
204
205
206
207
208
209
210
211
212
213
214
215
216
217
218
219
220
221
222
223
224
225
226
227
228
229
230
231
232
233
234
235
236
237
238
239
240
241
242
243
244
245
246
247
248
249
250
251
252
253
254
255
256
257
258
259
260
261
262
263
264
265
266
267
268
269
270
271
272
273
274
275
276
277
278
279
280
281
282
283
284
285
286
287
288
289
290
291
292
293
294
295
296
297
298
299
300
301
302
303
304
305
306
307
308
309
310
311
312
313
314
315
316
317
318
319
320
321
322
323
324
325
326
327
328
329
330
331
332
333
334
335
336
337
338
339
340
341
342
343
344
345
346
347
348
349
350
351
352
353
354
355
356
357
358
359
360
361
362
363
364
365
366
367
368
369
370
371
372
373
374
375
376
377
378
379
380
381
382
383
384
385
386
387
388
389
390
391
392
393
394
395
396
397
398
399
400
401
402
403
404
405
406
407
408
409
410
411
412
413
414
415
416
417
418
419
420
421
422
423
424
425
426
427
428
429
430
431
432
433
434
435
436
437
438
439
440
441
442
443
444
445
446
447
448
449
450
451
452
453
454
455
456
457
458
459
460
461
462
463
464
465
466
467
468
469
470
471
472
473
474
475
476
477
478
479
480
481
482
483
484
485
486
487
488
489
490
491
492
493
494
495
496
497
498
499
500
501
502
503
504
505
506
507
508
509
510
511
512
513
514
515
516
517
518
519
520
521
522
523
524
525
526
527
528
529
530
531
532
533
534
535
536
537
538
539
540
541
542
543
544
545
546
547
548
549
550
551
552
553
554
555
556
557
558
559
560
561
562
563
564
565
566
567
568
569
570
571
572
573
574
575
576
577
578
579
580
581
582
583
584
585
586
587
588
589
590
591
592
593
594
595
596
597
598
599
600
601
602
603
604
605
606
607
608
609
610
611
612
613
614
615
616
617
618
619
620
621
622
623
624
625
626
627
628
629
630
631
632
633
634
635
636
637
638
639
640
641
642
643
644
645
646
647
648
649
650
651
652
653
654
655
656
657
658
659
660
661
662
663
664
665
666
667
668
669
670
671
672
673
674
675
676
677
678
679
680
681
682
683
684
685
686
687
688
689
690
691
692
693
694
695
696
697
698
699
700
701
702
703
704
705
706
707
708
709
710
711
712
713
714
715
716
717
718
719
720
721
722
723
724
725
726
727
728
729
730
731
732
733
734
735
736
737
738
739
740
741
742
743
744
745
746
747
748
749
750
751
752
753
754
755
756
757
758
759
760
761
762
763
764
765
766
767
768
769
770
771
772
773
774
775
776
777
778
779
780
781
782
783
784
785
786
787
788
789
790
791
792
793
794
795
796
797
798
799
800
801
802
803
804
805
806
807
808
809
810
811
812
813
814
815
816
817
818
819
820
821
822
823
824
825
826
827
828
829
830
831
832
833
834
835
836
837
838
839
840
841
842
843
844
845
846
847
848
849
850
851
852
853
854
855
856
857
858
859
860
861
862
863
864
865
866
867
868
869
870
871
872
873
874
875
876
877
878
879
880
881
882
883
884
885
886
887
888
889
890
891
892
893
894
895
896
897
898
899
900
901
902
903
904
905
906
907
908
909
910
911
912
913
914
915
916
917
918
919
920
921
922
923
924
925
926
927
928
929
930
931
932
933
934
935
936
937
938
939
940
941
942
943
944
945
946
947
948
949
950
951
952
953
954
955
956
957
958
959
960
961
962
963
964
965
966
967
968
969
970
971
972
973
974
975
976
977
978
979
980
981
982
983
984
985
986
987
988
989
990
991
992
993
994
995
996
997
998
999
1000
\chapter{Jordan标准型}
\section{Jordan标准形:空间分解法}
\subsection{空间第二分解定理:循环不变子空间分解}
\begin{definition}[幂零线性变换]
$\mathcal{A}$是$V$内的线性变换,若$\exists m$使得$\mathcal{A}^m = 0$,
则$\mathcal{A}$是幂零线性变换。
\end{definition}
\begin{lemma}[幂零基]
$\mathcal{A}$是幂零变换,$\forall \alpha \in V$若$\mathcal{A}^{k-1}\alpha \neq 0, \mathcal{A}^k \alpha = 0$,
则$\alpha, \mathcal{A}\alpha , \cdots, \mathcal{A}^{k-1}\alpha$线性无关。
\end{lemma}
\begin{proof}
设$a_0 \alpha + a_1 \mathcal{A}\alpha + \cdots + a_{k-1} \mathcal{A}^{k-1}\alpha = 0$,
若从左至右第一个非零的是$a_i$,即$a_i \mathcal{A}^i \alpha + \cdots + a_k \mathcal{A}^{k-1} \alpha = 0$,
两侧同时作用$\mathcal{A}^{k-1-i}$,上式变为$a_i \mathcal{A}^{k-1}\alpha = 0$,
而$\mathcal{A}^{k-1}\alpha \neq 0$,故$a_i = 0$,与假设矛盾。
\end{proof}
\begin{definition}[循环不变子空间]
对$\forall \alpha \in V$,$\exists k$使得$\mathcal{A}^k = \mathbf{0}, \mathcal{A}^{k-1} \neq \mathbf{0}$,
则$I(\alpha) := L(\alpha, \mathcal{A}\alpha,\cdots, \mathcal{A}^{k-1}\alpha)$称为$\alpha$生成的$\mathcal{A}$的循环不变子空间。
\end{definition}
\begin{theorem}[循环不变子空间分解]
对于$V$内的幂零变换$\mathcal{A}$,$\exists v_1,\cdots,v_n \in V$,$m_1,\cdots,m_n \in \mathbb{N}$使得$V = I(v_1) \oplus I(v_2) \oplus \cdots \oplus I(v_n)$。
其中$I(v_i) = L( v_i, \mathcal{A}v_i \cdots, \mathcal{A}^{m_i} v_i)$,且$\mathcal{A}^{m_i + 1} v_i = 0$。
\end{theorem}
\begin{proof}
采用数归与核像分解。(1)若$dim V = 1$,则显然可分解。
(2)若$dim V > 1$,则用归纳法。
由于$\mathcal{A}$不是单射(因为将不同的向量均映射到$0$),因此$\mathcal{A}$不是满射(因为线性映射是满射当且仅当是双射),因此$\mathcal{A}$的像维数低于$dim V$。
对$Im \mathcal{A}$用归纳假设,得到$Im \mathcal{A}$的一组基$\mathcal{A}^{m_1} v_1,\cdots, \mathcal{A}v_1,v_1, \cdots, \mathcal{A}^{m_n}v_n,\cdots,v_n$,
由于上述向量组是$Im \mathcal{A}$的一组基,对$\forall j$存在$V$中向量$u_j$使得$v_j = \mathcal{A}u_j$。
下面说明下面的向量组线性无关:
\begin{equation} \label{equ:循环不变子空间分解1}
\mathcal{A}^{m_1 + 1}u_1, \cdots, \mathcal{A}u_1,u_1,\cdots, \mathcal{A}^{m_n+1}u_n ,\cdots,u_n
\end{equation}
原因:可设上述向量的一个线性组合为$0$,
两侧同时作用$\mathcal{A}$,则仅有$\mathcal{A}^{m_1+1}u_1, \cdots, \mathcal{A}^{m_n + 1}u_n$的系数可以非零(因为它们被作用后为0)。
去除其他项后,相当于设上面这组向量的某个线性组合等于$0$,由于它们也是$Im \mathcal{A}$基的一部分,彼此线性无关,所以它们的系数也必须为零。
于是将(\ref{equ:循环不变子空间分解1})扩成$V$的一组基$\mathcal{A}^{m_1 + 1}u_1,\cdots,u_1,\cdots, \mathcal{A}^{m_n+1}u_n ,\cdots, u_n, w_1, \cdots, w_p$,
由于$\mathcal{A} w_j \in Im \mathcal{A}$,因此存在(\ref{equ:循环不变子空间分解1})张成空间中元素$x_j$,使得$\mathcal{A} w_j = \mathcal{A} x_j$。
令$u_{n+j} = w_j - x_j$,则有$\mathcal{A} u_{n+j} = 0$,
从而(\ref{equ:循环不变子空间分解1})配上$u_{n+1},\cdots,u_{n+p}$是所需$V$的一组基。
\end{proof}
\subsection{核像升链与根子空间}
设$\mathcal{A}$是$V$上的线性变换,
定义$M_0 = \{0\}, M_i = \text{Ker} \mathcal{A}^i$,
$N_0 = V, N_i = \text{Im}( \mathcal{A}^i)$,
下面来研究它们的性质。
\begin{lemma}[核像空间升链] \label{lemma:M和N的包含关系}
上述空间有:$\{0\} = M_0 \subseteq M_1 \subseteq M_2 \subseteq \cdots$,
$V = N_0 \supseteq N_1 \supseteq N_2 \supseteq \cdots$
\end{lemma}
\begin{proof}
(1)对$\forall \alpha \in M_i$,由定义有$\mathcal{A}^i\alpha = 0$,
显然$\mathcal{A}^{i+1}\alpha =0$,因此$M_i \subseteq M_{i+1}$。
(2)对于$\alpha \in N_i$,由定义$\exists \beta \in V$使得$\alpha = \mathcal{A}^i \beta$,
从而$\alpha = \mathcal{A}^{i-1}( \mathcal{A} \beta) \in N_{i-1}$。
由$\alpha$任意性可知$N_i \subseteq N_{i-1}$。
\end{proof}
\begin{lemma}[核空间增长]
核空间$M_i$的增长满足以下性质:
\begin{itemize}
\item 若$M_k = M_{k+1}$,则$M_k = M_{k+1} = M_{k+2} = \cdots$
\item 若$dim V = n$,则$M_n = M_{n+1} = M_{n+2} = \cdots$。
\end{itemize}
\end{lemma}
\begin{proof}
(1)显然有$M_{k+1} \subseteq M_{k+2}$,只需要证明$M_{k+2} \subseteq M_{k+1}$。
对于$\forall \alpha \in M_{k+2}$,有$\mathcal{A}^{k+2}\alpha = 0$,即$\mathcal{A}^{k+1}( \mathcal{A}\alpha) = 0$,
因此$\mathcal{A} \alpha \in M_{k+1} = M_k$,即$\mathcal{A}^{k+1}\alpha = 0 \rightarrow \alpha \in M_{k+1}$,
由任意性可知$M_{k+2} \subseteq M_{k+1}$,故$M_{k+1} = M_{k+2}$,后续类似。
(2)只需要证明$M_n = M_{n+1}$,假设不然,
则$\{0\} = M_0 \subset M_1 \subset \cdots \subset M_n \subset M_{n+1}$,$dim M_{n+1} > n$,与$M_{n+1} \subseteq V$矛盾。
\end{proof}
\begin{theorem}[幂次变换核像分解] \label{thm:根子空间核像分解}
$V = M_n \oplus N_n$,其中$n = dim V$。
\end{theorem}
\begin{proof}
显然由核像性质有$dim M_n + dim N_n = n$,只需证$M_n \cap N_n = \{0\}$。
$\forall v \in M_n \cap N_n$,有$\mathcal{A}^n v = 0$,且$\exists u \in V$使得$v = \mathcal{A}^n u$,
从而$\mathcal{A}^{2n} u = 0$,而$M_{2n} = M_{2n-1} = \cdots = M_n$,
故$\mathcal{A}^n u = 0$,从而$v = 0$。
\end{proof}
\begin{definition}[广义特征向量与根子空间(广义特征子空间)]
\begin{itemize}
\item 广义特征向量:线性变换$\mathcal{A}$关于特征值$\lambda$的广义特征向量定义为满足$\exists j \in \mathbb{N}^+, ( \mathcal{A} - \lambda E)^j v = 0$的所有向量。
\item 根子空间:线性变换$\mathcal{A}$关于$\lambda$的所有广义特征向量加上零向量组成的空间称为$\mathcal{A}$关于特征值$\lambda$的根子空间,
记作$G(\lambda, \mathcal{A})$。
\end{itemize}
\end{definition}
\begin{theorem}[根子空间的具体表达] \label{lemma:根子空间表达式}
根子空间$G(\lambda, \mathcal{A})$满足$G(\lambda, \mathcal{A}) = \text{Ker}( \mathcal{A} - \lambda E)^{dim V}$。
\end{theorem}
\begin{proof}
右推左:若$v \in Ker( \mathcal{A} - \lambda E)^{dim V}$,则由$G(\lambda, \mathcal{A})$定义可知$v \in G(\lambda, \mathcal{A})$。
左推右:$v \in G(\lambda, \mathcal{A})$即$\exists j \in \mathbb{N} \text{ s.t. }(\mathcal{A} - \lambda E)^j v = 0$,
而前面已证明这样的$v$必然满足$v \in Ker( \mathcal{A} - \lambda E)^{dim V}$。
\end{proof}
\begin{note}
为了知识体系的完整,这里补充一下:$\lambda_i$根子空间的次数最低压缩为$\lambda_i$对应Jordan块的最高阶数,即最小多项式中的$\lambda_i$的阶数。
其他书也常用特征多项式的代数重数表示(题目中会遇到),
上面引理的dimV明显放缩过大。
\end{note}
\subsection{空间第一分解定理:根子空间分解}
\begin{theorem}[根子空间的不变性]
根子空间$G(\lambda, \mathcal{A})$是$\mathcal{A}$不变的。
\end{theorem}
\begin{proof}
由定理\ref{lemma:根子空间表达式}可知$G(\lambda, \mathcal{A}) = Ker( \mathcal{A} - \lambda E)^n$,
$\forall \alpha \in G(\lambda, \mathcal{A})$有$( \mathcal{A} - \lambda E)^n \alpha = 0$,
显然$( \mathcal{A} - \lambda E)^n \mathcal{A} \alpha = \mathcal{A}( ( \mathcal{A} - \lambda E)^n \alpha) = 0$,故$\mathcal{A} \alpha \in G(\lambda, \mathcal{A})$。
\end{proof}
\begin{theorem}[空间第一分解定理:根子空间分解]
若$V$是复线性空间,$\mathcal{A}$是线性变换,$\lambda_1,\cdots,\lambda_m$是其特征值,
则$V = G(\lambda_1, \mathcal{A}) \oplus \cdots \oplus G(\lambda_m, \mathcal{A})$。
\end{theorem}
\begin{proof}
用归纳法,(1)$n = 1$时显然成立(2)假设对所有小于$n$维空间均成立,由定理\ref{thm:根子空间核像分解}可知$V = G(\lambda_1, \mathcal{A}) \oplus Im ( \mathcal{A} - \lambda_1 E)^n$,
若$U = Im ( \mathcal{A} - \lambda_1 E)^n$为空,则显然成立。
其他情况首先显然$\mathcal{A}$在$U$上是不变的($\mathcal{A}$与$( \mathcal{A} - \lambda_1 E)$可交换),
去除$\lambda_1$所有对应特征向量后,
$\mathcal{A}|_U$在$U$有$\lambda_2,\cdots,\lambda_m$特征值,
对$U$用归纳法,$U = G(\lambda_2, \mathcal{A}|_U) \oplus \cdots \oplus G(\lambda_m, \mathcal{A}|_U)$,
由于$\forall j \geq 2, G(\lambda_j, \mathcal{A}|_U) = G(\lambda_j, \mathcal{A})$,
从而得证。
\end{proof}
\subsection{Jordan标准形:空间分解法}
% \begin{theorem}[上三角标准型]
% 若$V$是复线性空间,$\mathcal{A}$是$V$上的线性变换,
% 若$\lambda_1,\cdots,\lambda_m$是重数为$d_1,\cdots,d_m$的特征值,
% 则存在一组基使得$\mathcal{A}$在该基下的矩阵为
% $\left[
% \begin{array}{ccc}
% \lambda_j& & *\\
% & \ddots & \\
% 0& &\lambda_j
% \end{array}\right] _{d_j \times d_j}
% $组成的分块对角矩阵。
% \end{theorem}
\begin{theorem}[幂零变换的Jordan标准型]
幂零变换$\mathcal{A}$在循环不变子空间分解基下的矩阵如下。
称为幂零变换的Jordan标准形。
\end{theorem}
\begin{equation*}
J=\left[\begin{array}{cccc}
J_{1} & & & \\
& J_{2} & & 0 \\
0 & & \ddots & \\
& & & J_{s}
\end{array}\right], \quad J_{i}=\left[\begin{array}{cccc}
0 & 1 & & 0 \\
& 0 & \ddots & \\
& & \ddots & 1 \\
0 & & & 0
\end{array}\right]_{n_{i} \times n_{i}}
\end{equation*}
\begin{proof}
代入计算显然可得。
\end{proof}
\begin{theorem}[核限制变换是幂零变换]
对于$\forall$线性变换$\mathcal{A}$,
令$\mathcal{B} = \mathcal{A} - \lambda E$,
则$\mathcal{B}$在$M_k = Ker \mathcal{B}^k$中的限制变换$\mathcal{B}|_{M_k}$是幂零变换。
\end{theorem}
\begin{proof}
由于$M_k = \text{Ker} \mathcal{B}^k$,因此$\forall \alpha \in M_k$有$\mathcal{B}^k \alpha = 0$,
故$( \mathcal{B} |_{M_k})^k = 0$。
\end{proof}
\begin{theorem}[Jordan标准型:空间分解法]
对于$\mathbb{C}$上线性变换$\mathcal{A}$而言,
其有在先根子空间分解,根子空间再循环不变子空间分解的基下有以下形式。
称为线性变换的Jordan标准形。
\end{theorem}
\begin{equation*}
J=\left[\begin{array}{cccc}
J_{1} & & & 0 \\
& J_{2} & & \\
& & \ddots & \\
0 & & & J_{s}
\end{array}\right], \quad J_{i}=\left[\begin{array}{cccc}
\lambda_{i} & 1 & & 0 \\
& \lambda_{i} & \ddots & \\
& & \ddots & 1 \\
0 & & & \lambda_{i}
\end{array}\right]_{n_{i} \times n_{i}}
\end{equation*}
\begin{proof}
先由空间分解第一定理将$V$分解为根子空间直和,
由于$\mathcal{A} - \lambda E$在$G(\lambda, \mathcal{A})$中的限制变换是幂零变换,
因此再对$G(\lambda, \mathcal{A})$进行循环不变子空间分解,再由幂零变换的Jordan标准形可证。
\end{proof}
\section{空间分解法:零化多项式角度}
\subsection{根子空间分解:矩阵角度}
\begin{theorem}[解空间分解]
$A$是$n$阶方阵,
$f(x),f_1(x),f_2(x) \in P[x]$,
且$f(x) = f_1(x)f_2(x), (f_1(x),f_2(x)) = 1$,
设$f(A)X = 0, f_1(A)X = 0, f_2(A)X = 0$的解空间分别为$V,V_1,V_2$,
则
\begin{equation*}
V = V_1 \oplus V_2
\end{equation*}
\end{theorem}
\begin{proof}
由于互素,得到$\exists u(x),v(x)$,使得$E = u(A)f_1(A) + v(A) f_2(A)$,
从而$\forall \alpha \in V$有
\begin{equation*}
\alpha = u(A)f_1(A)\alpha + v(A) f_2(A)\alpha := \alpha_1 + \alpha_2
\end{equation*}
且满足$f(A)\alpha = f_1(A)f_2(A)\alpha = 0$。
故考察$f_2(A)\alpha_1$:
\begin{equation*}
f_2(A)\alpha_1 = f_2(A) u(A)f_1(A)\alpha = u(A)f_1(A)f_2(A) = 0 \Rightarrow \alpha_1 \in V_2
\end{equation*}
同理得到$f_1(A)\alpha_2 = 0, \alpha_2 \in V_1$,
从而得到$V = V_1 + V_2$。
下证直和,
$\forall \alpha \in V_1 \cap V_2$,
则$f_1(A)\alpha = f_2(A) \alpha = 0$,
因此
\begin{equation*}
\alpha = u(A)f_1(A) \alpha + v(A)f_2(A)\alpha = 0
\end{equation*}
因此结论成立。
\end{proof}
\begin{theorem}[多空间分解]
$A$是数域$P$上$n$阶方阵,
$f(x),f_1(x),\cdots, f_s(x) \in P[x]$,
$f(x) = f_1(x)f_2(x)\cdots f_s(x)$,
且$f_1(x),\cdots,f_s(x)$ \textbf{两两互素},
设$f(A)X = 0, f_1(A)X = 0,\cdots,f_s(A)X = 0$的解空间为$V,V_1,\cdots,V_s$,
则
\begin{equation*}
V = V_1 \oplus V_2 \oplus \cdots \oplus V_s
\end{equation*}
\end{theorem}
\begin{proof}
用数学归纳法即可。
\end{proof}
~
\begin{theorem}[空间分解研究矩阵秩]
考虑$f(A) = f_1(A)\cdots f_s(A)$,
则
\begin{equation*}
n - \mathrm{r}(f(A)) = n - \mathrm{r}(f_1(A)) + \cdots + n - \mathrm{r}(f_s(A))
\end{equation*}
\end{theorem}
\begin{proof}
设$f(A)X = 0, f_1(A)X = 0,\cdots,f_s(A) X = 0$的解空间为$V,V_1,\cdots,V_s$,
根据解空间分解得到$V = V_1 \oplus \cdots \oplus V_s$,
因此根据解空间的维数可得到结论。
\end{proof}
~
\begin{exercise}[空间分解研究矩阵的秩]
证明以下命题
(1)$A^2 = A$当且仅当$r(A) + r(A - E) = n$
(2)$A^2 = E$当且仅当$r(A+E) + r(A - E) = n$
(3)$A^3 = E$当且仅当$r(A - E) + r(A^2 + A + E) = n$
\end{exercise}
\begin{proof}
(1)$f(A) = A^2 - A = A(A - E)$,
因此$n - r(A^2 - A) = n - r(A) + n - r(A - E)$,
左侧等于$n$,右侧等于$2n - r(A) - r(A - E)$,
因此$r(A) + r(A - E) = n$
\end{proof}
~
\subsection{根子空间分解:线性变换角度}
\begin{lemma}[零变换的核空间]
$V$是$\mathbb{P}$上的线性空间,$\mathcal{O}$是零变换,则其核空间为整个空间:
\begin{equation*}
\mathrm{Ker}(\mathcal{O}) = V
\end{equation*}
\end{lemma}
\begin{theorem}[核空间分解]
$\mathcal{A}$为线性变换,
$f(x) = f_1(x)f_2(x)$为多项式,
且$(f_1(x),f_2(x)) = 1$,
则有如下核空间分解:
\begin{equation*}
\mathrm{Ker}(f(\mathcal{A})) = \mathrm{Ker}(f_1(\mathcal{A})) \oplus \mathrm{Ker}(f_2(\mathcal{A}))
\end{equation*}
\end{theorem}
\begin{proof}
根据$(f_1(x),f_2(x)) = 1$得到$\exists u(x),v(x)$使得$u(x)f_1(x) + v(x)f_2(x) = 1$,
因此
\begin{equation*}
\mathcal{E} = u(\mathcal{A})f_1(\mathcal{A}) + v(\mathcal{A}) f_2(\mathcal{A})
\end{equation*}
对$\forall \alpha \in \mathrm{Ker}(f(\mathcal{A}))$
\begin{equation*}
\alpha = \alpha_1 + \alpha _2 := u(\mathcal{A})f_1(\mathcal{A})\alpha + v(\mathcal{A})f_2(\mathcal{A})\alpha
\end{equation*}
显然$f_2(\mathcal{A})\alpha_1 = u(\mathcal{A})f(\mathcal{A})\alpha = 0$,
得到$\alpha_1 \in \mathrm{Ker}(f_1(\mathcal{A}))$,同理有$\alpha_2 \in \mathrm{Ker}(f_2(\mathcal{A}))$,综上得到
\begin{equation*}
\mathrm{Ker}(f(\mathcal{A})) = \mathrm{Ker}(f_1(\mathcal{A})) + \mathrm{Ker}(f_2(\mathcal{A}))
\end{equation*}
下证直和:对$\forall \alpha \in \mathrm{Ker}(f_1(\mathcal{A})) \cap \mathrm{Ker}(f_2(\mathcal{A}))$,有
\begin{equation*}
\alpha = u(\mathcal{A})f_1(\mathcal{A})\alpha + v(\mathcal{A})f_2(\mathcal{A})\alpha = 0
\end{equation*}
因此为直和
\end{proof}
\begin{theorem}[全空间分解:零化多项式角度]
$V$是$\mathbb{P}$上的线性空间,$\mathcal{A}$是$V$上的线性变换,
$f(x) = f_1(x) f_2(x) \cdots f_s(x)$为零化多项式,
且$f_1(x),\cdots,f_s(x)$两两互素,则
\begin{equation*}
V = \mathrm{Ker}(f_1(\mathcal{A})) \oplus \cdots \oplus \mathrm{Ker}(f_s(\mathcal{A}))
\end{equation*}
\end{theorem}
\begin{proof}
由于$f(A) = \mathcal{O}$,根据零变换的核空间以及核空间分解定理可知。
\end{proof}
\begin{lemma}[Hamilton-Cayley]
若$f(\lambda)$是线性变换$\mathcal{A}$的特征多项式,则$f(\lambda)$是$\mathcal{A}$的零化多项式
\end{lemma}
\begin{proof}
该定理在最小多项式一章已经证明过了
\end{proof}
\begin{theorem}[根子空间分解:零化多项式角度]
若$\mathbb{C}$上线性变换$\mathcal{A}$的特征多项式为$f(\lambda) = (\lambda - \lambda_1)^{k_1} \cdots (\lambda - \lambda_s)^{k_s}$,
最小多项式为$m(\lambda )= (\lambda - \lambda_1)^{l_1} \cdots (\lambda - \lambda_s)^{l_s}$,则
有以下根子空间分解
\begin{equation*}
V = \mathrm{Ker}(\mathcal{A} - \lambda_1 \mathcal{E})^{k_1}\oplus \cdots \oplus \mathrm{Ker}(\mathcal{A} - \lambda_s \mathcal{E})^{k_s} = \mathrm{Ker}(\mathcal{A} - \lambda_1 \mathcal{E})^{l_1} \oplus \cdots \oplus \mathrm{Ker}(\mathcal{A} - \lambda_s \mathcal{E})^{l_s}
\end{equation*}
\end{theorem}
\begin{proof}
根据Hamilton-Cayley定理以及零化多项式全空间分解可知。
\end{proof}
~
\begin{exercise}[根子空间分解:零化多项式角度]
(1)ZJU2021.8:$V$是线性空间,$\varphi$是$V$上线性变换,特征多项式$f(\lambda) = (\lambda - 2)^6(\lambda + 2)^4$,
将$V$分解为两个非平凡不变子空间的直和,并证明结论。
\end{exercise}
~
\begin{theorem}[根子空间分解与Jordan标准型]
根子空间分解将Jordan阵按照特征值$\lambda_i$分为若干份,每份由$\lambda_i$为对角的若干个Jordan块组成,
对角$\lambda_i$在Jordan阵中的出现次数等于其代数重数。
\end{theorem}
\subsection{Jordan阵与循环不变子空间}
\begin{lemma}[核限制变换是幂零变换]
$\lambda_i$为$\mathcal{A}$的特征值,则$(\mathcal{A} - \lambda_i \mathcal{E}) \big|_{\mathrm{Ker}(\mathcal{A} - \lambda_i \mathcal{E})^k}$在$\mathrm{Ker}(\mathcal{A} - \lambda_i \mathcal{E})^k$中为幂零变换
\end{lemma}
\begin{proof}
由于$\forall \alpha \in \mathrm{Ker}(\mathcal{A} - \lambda_i \mathcal{E})^k $满足
$(\mathcal{A} - \lambda_i\mathcal{E})^k \alpha = 0$,
因此限制变换满足$(\mathcal{A} - \lambda_i\mathcal{E})^k\big|_{\mathrm{Ker}(\mathcal{A} - \lambda_i\mathcal{E})^k} = \mathcal{O}$
\end{proof}
\begin{theorem}[循环不变子空间分解]
对于$V$内的幂零变换$\mathcal{A}$,$\exists v_1,\cdots,v_n \in V$,$m_1,\cdots,m_n \in \mathbb{N}$使得$V = I(v_1) \oplus I(v_2) \oplus \cdots \oplus I(v_n)$。
其中$I(v_i) = L( v_i, \mathcal{A}v_i \cdots, \mathcal{A}^{m_i} v_i)$,且$\mathcal{A}^{m_i + 1} v_i = 0$。
\end{theorem}
\begin{proof}
在空间第二分解定理中已经证明过了。
\end{proof}
\begin{theorem}[Jordan阵与循环不变子空间]
线性变换$\mathcal{A}$在根子空间$\mathrm{Ker}(\mathcal{A} - \lambda_i\mathcal{E})^k$下的限制变换$\mathcal{A} \big|_{\mathrm{Ker}(\mathcal{A} - \lambda_i \mathcal{E})^k}$在一组循环不变基$((\mathcal{A} - \lambda_i\mathcal{E})^s\alpha, (\mathcal{A} - \lambda_i \mathcal{E})^{s-1}\alpha,\cdots,\alpha)$下的矩阵为一个Jordan块:
\begin{equation*}
J_s(\lambda_i) = \left[
\begin{array}{ccccc}
\lambda_i&1&&& \\
&\lambda_i&1&&\\
&&\ddots&\ddots& \\
&&&\lambda_i&1\\
&&&&\lambda_i
\end{array}
\right]
\end{equation*}
\end{theorem}
\begin{proof}
直接验证即可。
\end{proof}
\begin{theorem}[Jordan阵与几何重数]
特征值$\lambda_i$的代数重数是特征多项式$f(\lambda)$的$\lambda = \lambda_i$根的重数,
几何重数($\lambda_i E - A = 0$解空间维数)是$\lambda_i$对应以$\lambda_i$为对角的Jordan块个数。
\end{theorem}
\begin{proof}
根据$\lambda_i E - A = 0$的解空间维数等于$\lambda_i E - J = 0$解空间维数,
而每有一个以$\lambda_i$为对角的Jordan块,$\lambda_i E - J$上就有一行全为$0$,
因此秩减$1$,从而块数等于几何重数。
\end{proof}
\begin{note}
即以$\lambda_i$为对角的Jordan块个数为$n - r(\lambda E - A)$,
反过来写也要分得清。
\end{note}
\section{Jordan标准形:$\lambda$矩阵法}
\subsection{$\lambda$矩阵与相抵}
\begin{definition}[$\lambda$矩阵]
$A(\lambda)$是元素为$\lambda$多项式的矩阵,则将$A(\lambda)$称为$\lambda$矩阵。
\end{definition}
\begin{definition}[$\lambda$矩阵初等变换]
$\lambda$矩阵有三种初等变换:
\begin{itemize}
\item 互换两行/列
\item 用常数c乘一行/列(不能乘$\lambda$多项式!)
\item 将一行/列乘以一个$\lambda$多项式加到另一行/列上
\end{itemize}
\end{definition}
\begin{definition}[相抵]
若可以用初等变换将一个$\lambda$矩阵转换为另一个$\lambda$矩阵,
则称两个$\lambda$矩阵相抵。
\end{definition}
\begin{theorem}[相抵标准形:法式]
对于数值矩阵$A$,$\lambda E - A$总是相抵于$\text{diag}\{1,\cdots,1,d_1(\lambda), \cdots, d_m(\lambda)\}$,
其中$d_i(\lambda)$是首一多项式,且$d_i(\lambda)|d_{i+1}(\lambda)$。
\end{theorem}
\begin{lemma}[对角整除化]
$A(\lambda)$为非零$\lambda$矩阵,
则$A(\lambda)$一定相抵于$B(\lambda)$,
满足$b_{11}(\lambda) \neq 0$且$b_{11}(\lambda)$可整除$B(\lambda)$中的所有元素
\end{lemma}
\begin{proof}
先将$A(\lambda)$中次数最低的元素放置于对角$a_{11}(\lambda)$
(1)消去第一行、第一列:不妨设$a_{i1}(\lambda)$不被$a_{11}(\lambda)$整除,
则做带余除法$a_{i1}(\lambda) = q(\lambda)a_{11}(\lambda) + r(\lambda)$,
再将$r(\lambda)$放置于$a_{11}$,直至全部整除,可消去第一列和第一行。
(2)考虑第一列、第一行外的其他元素,若存在$a_{ij}(\lambda)$不被$a_{11}$整除,
则把第$j$列加到第一列上,做带余除法,把余项放到$a_{11}$,以此类推。
\end{proof}
\begin{theorem}[法式的计算:初等变换法]
每次把最低次移动到角上,尝试消除,消除过程按照以下步骤
\begin{itemize}
\item 通过初等变换将当前对角元素变为所有非零元素中次数最低的$\lambda$多项式
\item 将矩阵对角整除化:具体见引理的过程
\item 消去一列,再消去一行,重复对角整除化和消去
\end{itemize}
\end{theorem}
\begin{theorem}[相抵与相似]
$A$与$B$相似当且仅当$\lambda E - A$与$\lambda E - B$相抵。
\end{theorem}
\begin{proof}
(1)左推右:若$B = P^{-1}AP$,则$\lambda E - B = P^{-1}(\lambda E - A)P$,
显然右侧两个$\lambda$矩阵相抵。
(2)右推左:若$\lambda E - B = P(\lambda)(\lambda E - A)Q(\lambda)$,
设$Q(\lambda) = Q_k\lambda^k + \cdots + Q_1\lambda + Q_0$,
$Q(\lambda)^{-1} = R_m\lambda^m + \cdots + R_1\lambda + R_0$,
下记$W = Q(B)$。由$Q(\lambda) Q^{-1}(\lambda) = E$得
$R_mQ(\lambda)\lambda^m + \cdots + R_1Q(\lambda)\lambda + R_0Q(\lambda) = E$,
代入$B$得到$R_mWB^m + \cdots + R_1WB + R_0W = E$。
由于$P(\lambda)^{-1}(\lambda E - B) = (\lambda E - A)Q(\lambda) = Q(\lambda)\lambda - A Q(\lambda)$,代入$\lambda = B$得到$Q(B)B = AQ(B)$(前面一式子左=$0$),
即$WB = AW$,从而$WB^l = A^lW$(多作用一个$A$相当于多右乘一个$B$)。
再由前面的式子得到$(R_mA^m + \cdots + R_0)W = E$,则表明$W$可逆,
从而$B = W^{-1}AW$。
\end{proof}
\subsection{行列式因子、不变因子}
\begin{definition}[行列式$k$阶子式]
$|A|$的$k$阶子式是其任选$k$行、$k$列组成的矩阵的行列式。
\end{definition}
\begin{definition}[行列式因子]
$A(\lambda)$是$\lambda$矩阵,若其所有$k$阶子式(行列式)均为$0$,
则定义其$k$阶行列式因子$D_k(\lambda) = 0$,
否则定义为所有非零$k$阶子式的首一最大公因式。
\end{definition}
\begin{note}
一般行列式因子比初等变换方便,不过一定注意这里的子式是所有子式,
而非主子式,甚至可以不连着,$0$的情况也得分开考虑。
\end{note}
\begin{lemma}[行列式因子的相除性]
设$D_1(\lambda),\cdots,D_r(\lambda)$是$A(\lambda)$非零行列式因子,
则$D_i(\lambda) | D_{i+1}(\lambda)$。
\end{lemma}
\begin{proof}
设$A_{i+1}$是$A(\lambda)$任意$i+1$阶子式,
这个行列式按某一行展开,则其每一展开项都是一个多项式与一个$i$阶子式乘积,
而$D_i(\lambda)$是所有$i$阶子式公因子,因此$D_i(\lambda) | A_{i+1}(\forall i)$,
而$D_{i+1}(\lambda)$是所有$A_{i+1}$的最大公因式,因此证毕。
\end{proof}
\begin{definition}[不变因子]
定义$d_1(\lambda ) = D_1(\lambda)$,$d_i(\lambda) = \frac{D_i(\lambda)}{D_{i-1}(\lambda)}(\forall i \neq 1)$为$A(\lambda)$的不变因子。
\end{definition}
\begin{theorem}[行列式因子、不变因子的性质]
\begin{itemize}
\item 相抵矩阵的因子:相抵的$\lambda$矩阵有相同的行列式因子、不变因子(初等变换不改变行列式、不变因子)
\item 法式与不变因子:
$\lambda E - A$法式的所有对角元素即$\lambda E - A$的不变因子。
\end{itemize}
\end{theorem}
\begin{proof}
(1)只需证明行列式因子在三种初等变换不变即可。
而不变因子由行列式因子唯一确定。
(2)由于不变因子在初等变化中不变,求出法式的不变因子(即对角),因此$\lambda E - A$的不变因子即其法式对角元素。
\end{proof}
\begin{theorem}[不变因子与最小多项式]
$A$的最小多项式即$\lambda E - A$的最后一个不变因子
\end{theorem}
\begin{proof}
设$A$的互异特征值为$\lambda_1,\cdots,\lambda_s$,
因此可以列出所有的初等因子(具体见下一节),
最小多项式即初等因子的最小公倍式,
最后一个不变因子也是,
因此结论成立
\end{proof}
\subsection{初等因子与Jordan标准型}
\begin{definition}[初等因子]
将所有不变因子分解为不可约因子之积,这些不可约因子全体称为初等因子组。
\end{definition}
\begin{theorem}[对角化计算初等因子] \label{lemma:对角化计算初等因子}
$\lambda E - A$经过初等变化变为对角阵(不一定要法式)
\begin{equation*}
\left(\begin{array}{llll}
f_{1}(\lambda) & & & \\
& f_{2}(\lambda) & & \\
& & \ddots & \\
& & & f_{n}(\lambda)
\end{array}\right)
\end{equation*}
其中$f_i(\lambda)$为非零首一多项式,则$A$的初等因子组是所有$f_i(\lambda)$做不可约分解得到的因式组合。
\end{theorem}
\begin{lemma}[Jordan块、Jordan阵的初等因子]
$r$阶$\lambda_0$对角的Jordan块的初等因子组为$(\lambda - \lambda_0)^r$。
对角为$J_1,J_2,\cdots,J_k$的Jordan阵的初等因子组为$(\lambda - \lambda_1)^{r_1}, \cdots, (\lambda - \lambda_k)^{r_k}$。
\end{lemma}
\begin{proof}
(1)Jordan块的初等因子组直接用行列式因子计算即可(只有$r$阶非$1$,因此不变因子好算)
(2)Jordan阵的初等因子进行分块初等变换,将各个Jordan块分块变成法式,
再用引理\ref{lemma:对角化计算初等因子}即可。
\end{proof}
\begin{theorem}[使用初等因子计算Jordan标准型]
若$K$上的矩阵$A$有初等因子组$(\lambda - \lambda_1)^{r_1}, \cdots, (\lambda - \lambda_k)^{r_k}$,
则$A$相似于Jordan阵$J = \text{diag}\{J_1,\cdots,J_k\}$,
其中$J_i$是以$\lambda_i$为对角的$r_i$阶Jordan块。
\end{theorem}
\begin{theorem}[Jordan标准型存在充要条件]
$K$上矩阵$A$有Jordan标准型当且仅当其初等因子都是一次项的幂次乘积。
\end{theorem}
\section{$\lambda$矩阵的进一步研究}
\subsection{Jordan标准型的直接计算}
一、初等变换方法:只需要将$\lambda E - A$转换为对角矩阵,对角所有分解即初等因子组,直接构造Jordan标准型
~
\begin{exercise}[无法直接整除的情况]
(1)计算下面矩阵$A$的法式:
\begin{equation*}
A = \left[
\begin{array}{ccc}
-1&1&3 \\
3&0&-4 \\
-2&1&4
\end{array}
\right]
\end{equation*}
\end{exercise}
\begin{solution}
(1)进行到下面步骤的时候无法整除,即发现不整除列上的多项式,要进行对角整除化
\begin{equation*}
\left[
\begin{array}{ccc}
1&0&0 \\
0&\lambda-1&-\lambda+1 \\
0&\lambda^2 + \lambda - 3&-3\lambda + 4
\end{array}
\right] \rightarrow \left[
\begin{array}{ccc}
1&0&0 \\
0&\lambda - 1&- \lambda + 1 \\
0&(\lambda^2 + \lambda - 3) - (\lambda-1)(\lambda+2)&(-3\lambda + 4) - (\lambda+2)(-\lambda + 1)
\end{array}
\right] = \left[
\begin{array}{ccc}
1&0&0 \\
0&\lambda - 1&-\lambda + 1 \\
0&-1&\lambda^2 - 2\lambda +2
\end{array}
\right]
\end{equation*}
因此最终可以化简为:
\begin{equation*}
\left[
\begin{array}{ccc}
1&& \\
&1& \\
&&(\lambda-1)^3
\end{array}
\right]
\end{equation*}
\end{solution}
~
\begin{exercise}[初等变换方法计算Jordan标准型]
计算$A = \left[
\begin{array}{ccc}
1&-1&2\\
3&-3&6\\
2&-2&4
\end{array}
\right]$的Jordan标准型
\end{exercise}
\begin{solution}
通过初等变换得到$\lambda E - A$的法式为$\left[
\begin{array}{ccc}
1&0&0\\
0&\lambda&0\\
0&0&-\lambda^2 + 2\lambda
\end{array}
\right]$,
因此初等因子为$\lambda,\lambda,(\lambda - 2)$,
对应Jordan阵为:
\begin{equation*}
J = \left[
\begin{array}{ccc}
0&& \\
&0& \\
&&2
\end{array}
\right]
\end{equation*}
\end{solution}
~
二、行列式因子方法:求一般矩阵的Jordan阵用行列式因子可能比较麻烦,但是对于抽象的$n$阶矩阵却有很大的作用,
原因在于只要有两个子式互素,则行列式因子就为$1$。
~
\begin{exercise}[经典而重要的题目]
$a,b \in \mathbb{C}$,求下面矩阵$A$的Jordan标准型
\begin{equation*}
A = \left[
\begin{array}{ccccc}
a&b&\cdots&b&b \\
&a&\cdots&b&b \\
&&\ddots&\vdots&\vdots \\
&&&a&b \\
&&&&a
\end{array}
\right]
\end{equation*}
\end{exercise}
\begin{solution}
若$b = 0$,则Jordan标准型即$aE$。
若$b \neq 0$,则考虑$\lambda E - A$:
\begin{equation*}
\lambda E - A =
\left[
\begin{array}{ccccc}
\lambda - a&-b&\cdots&-b&-b \\
&\lambda - a&\cdots&-b&-b \\
&&\ddots&\vdots&\vdots \\
&&&\lambda - a&-b \\
&&&&\lambda - a
\end{array}
\right]
\end{equation*}
考虑$n-1$阶子式,右下子式行列式为$(\lambda - a)^{n-1}$,
右上子式在$\lambda = a$时显然不是$0$(根据余数定理),
因此互素,即初等因子为$(\lambda - a)^n$,因此Jordan阵为一个$n$阶的以$a$为对角的Jordan块
\end{solution}
~
\begin{exercise}[行列式因子应用相关练习]
求不变因子:
(1)$A = \left[
\begin{array}{cccc}
&&1&\lambda+2 \\
&1&\lambda+2& \\
1&\lambda+2&& \\
\lambda+2&&&
\end{array}
\right]$
(2)$A = \left[
\begin{array}{cccc}
\lambda&-1&& \\
&\lambda&-1& \\
&&\lambda&-1 \\
5&4&3&\lambda+2
\end{array}
\right]$
\end{exercise}
\begin{solution}
(1)左上行列式因子为$-1$,
因此不变因子为$1,1,1,(\lambda+2)^4$
(2)右上行列式因子为$-1$,
因此不变因子$1,1,1,|A|$
\end{solution}
~
\begin{theorem}[重要$\lambda$矩阵]
下面$\lambda$矩阵$A$的不变因子为$1,1,\cdots,\lambda^n + a_1\lambda^{n-1}+a_2\lambda^{n-2} + \cdots + a_{n-1}\lambda + a_n$
\begin{equation*}
A = \left[
\begin{array}{cccccc}
\lambda&&&&&a_n \\
-1&\lambda&&&&a_{n-1} \\
&-1&\lambda&&&a_{n-2} \\
&&\ddots&\ddots&&\vdots \\
&&&-1&\lambda&a_2 \\
&&&&-1&\lambda+a_1
\end{array}
\right]
\end{equation*}
\end{theorem}
\begin{proof}
考虑左下方行列式因子,其值为$1$,因此不变因子为$1,1,\cdots,1,|A|$,
而计算出$|A| = \lambda^n + a_1 \lambda^{n-1} + \cdots + a_n $,因此结论成立
\end{proof}
\begin{corollary}
下列矩阵$A$在$\mathbb{C}$可对角化当且仅当$\lambda^n + a_1\lambda^{n-1}+a_2\lambda^{n-2} + \cdots + a_{n-1}\lambda + a_n$在$\mathbb{C}$上无重根
\begin{equation*}
A = \left[
\begin{array}{cccccc}
0&&&&&a_n \\
-1&0&&&&a_{n-1} \\
&-1&0&&&a_{n-2} \\
&&\ddots&\ddots&&\vdots \\
&&&-1&0&a_2 \\
&&&&-1&a_1
\end{array}
\right]
\end{equation*}
\end{corollary}
~
\begin{exercise}[一些用行列式因子计算方便的特殊矩阵]
计算Jordan标准型
(1)$A = \left[
\begin{array}{cccc}
1&2&3&4 \\
&1&2&3 \\
&&1&2 \\
&&&1
\end{array}
\right]$
(2)$A = \left[
\begin{array}{cccc}
a&&& \\
a&a&& \\
\vdots&\vdots&\ddots& \\
a&a&\cdots&a
\end{array}
\right]$
(3)$A = \left[
\begin{array}{cccc}
a&&& \\
b_1&a&& \\
&\ddots&\ddots& \\
&&b_{n-1}&a
\end{array}
\right]$
(4)$A = \left[
\begin{array}{ccccc}
a&a_{12}&a_{13}&\cdots&a_{1n} \\
&a&a_{23}&\cdots&a_{2n} \\
&&\ddots&&\vdots \\
&&&a&a_{n-1,n} \\
&&&&a
\end{array}
\right]$,
$a_{12}a_{23}\cdots a_{n-1,n} \neq 0$
\end{exercise}
\begin{solution}
(1)$\lambda E - A$左上行列式为$(\lambda - 1)^3$,
右上在$\lambda = 1$是显然非零,因此不变因子$1,1,1,|\lambda E - A|$
(2)左上行列式$(\lambda - a)^{n-1}$,
左下在$\lambda = a$显然非零,因此不变因子$1,1,\cdots,1,|\lambda E - A|$
(3)(4)显然
\end{solution}
\begin{note}
如果对角相等,则用行列式因子计算可能会比较方便
\end{note}
\subsection{$k$阶Jordan块的个数}
\begin{theorem}[$k$阶Jordan块个数]
$n$阶复矩阵$A$的Jordan标准型为$J$,
则$J$中对角元$\lambda$的$k$阶Jordan块$J_k(\lambda)$的个数如下。这里约定$r((A - \lambda E_n)^0) = n$
\begin{equation*}
r((A - \lambda E_n)^{k-1}) - 2r((A - \lambda E_n)^k) + r((A - \lambda E_n)^{k+1})
\end{equation*}
\end{theorem}
\begin{proof}
考虑零对角Jordan块的幂次:
\begin{equation*}
r(J_r(0)^k) =
\begin{cases}
r - k, & 0 \leq k < r\\
0, & k \geq r
\end{cases} \quad
r(J_r(0)^k) - r(J_r(0)^{k+1}) =
\begin{cases}
1, & 0 \leq k < r\\
0, & k \geq r
\end{cases}
\end{equation*}
而非零对角Jordan块的幂次满足
$r(J_r(a)^k) - r(J_r(a)^{k+1}) = 0$。
设$A$的Jordan标准型为$J = \text{diag}\{J_{r_1}(\lambda_1),\cdots,J_{r_s}(\lambda_s)\}$,
$(A - \lambda E)^k, (J - \lambda E)^k$相似。
根据$J$是分块对角的性质可知$r((A - \lambda E)^k) = r((J - \lambda E)^k) = \sum\limits_{i = 1}^s r(J_{r_i}(\lambda_i - \lambda)^k)$,
因此得到:
\begin{equation*}
r((A - \lambda E)^k) - r((A - \lambda E)^{k+1}) = \sum\limits_{i = 1}^s [r(J_{r_i}(\lambda_i - \lambda)^k) - r(J_{r_i}(\lambda_i - \lambda)^{k+1})]
\end{equation*}
根据前面的分析,$r((A - \lambda E)^k) - r((A - \lambda E)^{k+1})$等于$\lambda_i = \lambda$的且阶数大于$k$那些块个数,
同理$r((A - \lambda E)^{k-1}) - r((A - \lambda E)^k)$等于对角为$\lambda$的且阶数大于$k - 1$的块个数,
相减即可得到阶数为$k$的块个数。
\end{proof}
~
\begin{exercise}[上述定理的应用]
计算$A$的Jordan标准型
\begin{equation*}
A = \left[
\begin{array}{ccccc}
a&0&1&& \\
&a&0&\ddots& \\
&&a&\ddots&1 \\
&&&\ddots&0 \\
&&&&a
\end{array}
\right]
\end{equation*}
\end{exercise}
~
\begin{exercise}
$A,B$是两个$n$阶矩阵,证明$A,B$相似的充要条件是$\forall \lambda \in \mathbb{C}$,$k = 1,2,\cdots,n$,
均有$r((A - \lambda E)^k) = r((B - \lambda E)^k)$
\end{exercise}
\begin{proof}
直接用结论即可
\end{proof}
\subsection{分块矩阵的Jordan阵}
\begin{theorem}[分块矩阵的Jordan标准型]
$P$上$n$阶矩阵$A,B$无公共复特征值,
且$A,B$的Jordan标准型分别为$J_1,J_2$,则对任意$C \in P^{m \times n}$,
则:
\begin{equation*}
\left[
\begin{array}{cc}
A&C\\
O&B
\end{array}
\right] \sim \text{diag}\{J_1,J_2\}
\end{equation*}
\end{theorem}
\begin{proof}
当$A,B$无公共特征值时,根据$AX - XB$的矩阵方程结论可知,